• YOU can help the next generation of students in the community!
    Share your trial papers and notes on our Notes & Resources page

Search results

  1. Sy123

    HSC 2015 MX1 Marathon (archive)

    Re: HSC 2015 3U Marathon This is simply a completion of kawaiipotato's answer, starting from the bold (*) here:
  2. Sy123

    HSC 2015 MX1 Marathon (archive)

    Re: HSC 2015 3U Marathon As kawaiipotato has proven, one needs to only prove: \frac{(k+1)^{k+1}}{2^k} \leq \left(\frac{k+2}{2} \right)^{k+1} We can re-arrange it and cancel out the denominators, to proving: \\ $Prove that$ \ \left(\frac{k+2}{k+1} \right)^{k+1} > 2 \\ $So,$ \...
  3. Sy123

    HSC 2015 MX1 Marathon (archive)

    Re: HSC 2015 3U Marathon It wouldn't receive full marks unless you prove that last line as it isn't an immediate result I'll post my solution soon
  4. Sy123

    HSC 2015 MX1 Marathon (archive)

    Re: HSC 2015 3U Marathon The addition here is the wrong way around
  5. Sy123

    HSC 2015 MX1 Marathon (archive)

    Re: HSC 2015 3U Marathon \\ $i) Using the definition$ \ \binom{n}{r} := \frac{n!}{(n-r)!r!} \\ $So,$ \ \binom{n}{r} = \frac{n!}{(n-r)! r!} = \frac{n!}{(n- (n-r))! (n-r)!} = \binom{n}{n-r} \\ $ii) Transform the expression$ \ f(r) \ $into an easier to deal form, that is, replace every$ \...
  6. Sy123

    HSC 2015 MX1 Marathon (archive)

    Re: HSC 2015 3U Marathon On track up to this part, your proof of this is not valid Just because a \geq b \ $and$ \ c \geq d does not mean \frac{a}{c} \geq \frac{b}{d} (which is what you do when you divide the inequalities (k+2) > 3 and (k+1) > 2) For instance, 4 > 3 and 3 > 2 but...
  7. Sy123

    HSC 2016 MX2 Marathon ADVANCED (archive)

    Re: HSC 2016 4U Marathon - Advanced Level 2014 (actually 2013) stopped because mods wanted it to (I wanted it to keep going)
  8. Sy123

    HSC 2016 MX2 Marathon ADVANCED (archive)

    Re: HSC 2016 4U Marathon - Advanced Level \\ $Show that the only pairs of positive integers$ \ x , y \ $so that$ \\ x^y = y^x \ $and$ \ x \neq y \\ $are$ \ x = 2, y = 4 \ $and$ \ x = 4, y = 2
  9. Sy123

    HSC 2016 MX2 Marathon ADVANCED (archive)

    Post questions within the scope of Mathematics Extension 2 that are in general Q16 and beyond, focusing on problem solving and neat results within the reach of elementary mathematics.
  10. Sy123

    HSC 2015 MX1 Marathon (archive)

    Re: HSC 2015 3U Marathon \\ $Prove by induction or otherwise for integers$ \ n \geq 1 \ $that$ \ n! \leq \left(\frac{n+1}{2} \right)^n
  11. Sy123

    HSC 2015 MX1 Marathon (archive)

    Re: HSC 2015 3U Marathon \\ 3^4 > 4^3 \ $so it is true for$ \ n = 3 \\ $Assume true for$ \ n = k \Rightarrow k^{k+1} > (k+1)^k \\ $Using the inequality from the assumption, we get$ \\\\ \frac{k^{k+1}}{(k+1)^k} (k+2)^{k+1} > (k+2)^{k+1} \\ k(k+2) < (k+1)^2 \Rightarrow k^{k+1} (k+1)^{k+1}...
  12. Sy123

    Maths Extension 2 thoughts

    Anyone with the paper please PM me
  13. Sy123

    HSC 2015 MX2 Marathon (archive)

    Re: HSC 2015 4U Marathon Good luck MX2 students!
  14. Sy123

    HSC 2015 MX2 Marathon (archive)

    Re: HSC 2015 4U Marathon \\ $i)$ \ P_1 = \frac{1}{2} = \frac{c_1}{d_1} = \frac{\text{odd}}{\text{even}} \ \Rightarrow \ $true for$ \ n = 1 \\ $Assume true of$ \ P_k = \frac{c_k}{d_k} \\ P_{k+1} = P_k + \frac{1}{p_{k+1}} = \frac{c_k p_{k+1} + d_k}{d_k p_{k+1}} \\ $Since$ \ p_{k+1} \ $is...
  15. Sy123

    HSC 2015 MX2 Marathon (archive)

    Re: HSC 2015 4U Marathon \\ $Consider the sum$ \ P_n = \sum_{i=1}^n \frac{1}{p_i} \ $where$ \ p_i \ $is the$ \ i^{th} \ $prime number, so$ \\ p_1 = 2, \ p_2 = 3, \ p_3 = 5, \ $etc.$ \\ $i) Prove by mathematical induction that$ \ P_n \ $when simplified into a single fraction, is$ \ P_n =...
  16. Sy123

    HSC 2015 MX2 Marathon (archive)

    Re: HSC 2015 4U Marathon For a HSC solution without invoking AM-GM: \\ $Let$ \ A = x_1 + x_2 + \dots + x_{n-1} \ $and$ \ B = x_1 x_2 \dots x_{n-1} \\ $The inequality to be proven then becomes:$ \ \left(\frac{A + x_n}{n \sqrt[n]{Bx_n}} \right)^n \geq \left(\frac{A}{(n-1)\sqrt[n-1]{B}}...
  17. Sy123

    HSC 2015 MX2 Marathon (archive)

    Re: HSC 2015 4U Marathon \\ $Prove for positive reals$ \ x_1,x_2, \dots , x_n \\ \\ \left(\frac{x_1 + x_2 + \dots + x_n}{n \sqrt[n]{x_1x_2\dots x_n}}\right)^n \geq \left( \frac{x_1 + x_2 + \dots + x_{n-1}}{(n-1) \sqrt[n-1]{x_1 x_2\dots x_{n-1}}} \right)^{n-1}
  18. Sy123

    Struggling with inequalities.

    Another tip I can give, is to use all resources of mathematical knowledge at your disposal. For example, using calculus, one can in fact prove: a^3+ b^3+c^3 \geq 3abc After, all just look at the function f(x) = \frac{1}{3} (x^3 + (b^3 + c^3)) - x (3bc) , \ x > 0 And all you need to do is...
  19. Sy123

    HSC 2015 MX2 Marathon (archive)

    Re: HSC 2015 4U Marathon Better safe than sorry
  20. Sy123

    HSC 2015 MX2 Marathon (archive)

    Re: HSC 2015 4U Marathon Yes, or a more intuitive way of looking at it: x \geq \ln (1+x) \frac{1}{k} \geq \ln \left( 1 + \frac{1}{k} \right) \frac{1}{k} \geq \ln \left(\frac{1+k}{k} \right) = \ln ( k+1) - \ln k \sum_{k=1}^{n} \frac{1}{k} > \ln 2 - \ln 1 + \ln 3 - \ln 2 + \dots + \ln...
Top